Sydney measured a line to be 4.9 inches long. If the actual length of the line is 5.2 inches, then what was the percent error of the measurement, to the nearest tenth of a percent?

Answers

Answer 1

The percent error of the measurement will be 5.8%.

What is mean by Percentage?

A number or ratio that can be expressed as a fraction of 100 or a relative value indicating hundredth part of any quantity is called percentage.

To Calculate the percent of a number , divide the number by whole number and multiply by 100.

Given that;

Sydney measured a line to be 4.9 inches long.

And, The actual length of the line is 5.2 inches.

Now,

The percent error is calculated as;

The difference between the measures = 5.2 - 4.9

                                                            = 0.3 inches

So, The percent error = 0.3/5.2 x 100

                                  = 5.76%

Thus, The percent error of the measurement to the nearest tenth of a percent will be 5.8%.

Learn more about the percent visit:

https://brainly.com/question/24877689

#SPJ1


Related Questions

which of the following is the largest? half of 78a third of 114one-fifth of 190

Answers

Let's get the following value for the given statements and check which are the largest.

a) Half of 78.

We compute for the half of 78, which we divide 78 by 2. We have

[tex]\frac{78}{2}=39[/tex]

b) A third of 114

A third of a number means we divide the number by 3. Dividing 114 by 3, we get

[tex]\frac{114}{3}=38[/tex]

c) One-fifth of 190

One-fifth of a number means we divide the number by 3. Dividing 190 by 5, we get

[tex]\frac{190}{5}=38[/tex]

As we can see on the results above, the largest is half of 78.

a store has 24 saltwater fish. the store has 4 tanks for the fish. each tank has an equal number of fish. how many fish are in each tank

Answers

Answer:

Step-by-step explanation:

Answer: 6

there are 24 fish and 4 tanks so divide 24 by 4 and you'll have 6

Answer:

the number of fish in each tank is 6

Step-by-step explanation:

becuause 24 divided be 4 is 6

use the unit circle to find sec(7/6)

Answers

Step 1

Draw the unit circle required

Step 2

Find the value sec(7π/6) in cosine

[tex]\begin{gathered} \sec (\frac{7\pi}{6})=\frac{1}{cos(\frac{7\pi}{6})} \\ \sec (x)=\frac{1}{cos(x)} \end{gathered}[/tex]

Step 3

Find cos(7π/6)

The trigonometric unit circle and a trigonometric table gives;

[tex]\begin{gathered} \cos (\frac{7\pi}{6})=\cos (\frac{\pi}{6}+\pi) \\ \cos (\frac{7\pi}{6})=\text{cos}(\frac{\pi}{6})\cos (\pi)-\sin (\frac{\pi}{6})sin\pi=-\cos (\frac{\pi}{6}) \\ \cos (\frac{7\pi}{6})=\frac{\sqrt[]{3}}{2}(-1)-(\frac{1}{2})(0)=-\frac{\sqrt[]{3}}{2} \\ \cos (\frac{7\pi}{6})=-\frac{\sqrt[]{3}}{2} \end{gathered}[/tex]

Step 4

Find sec(7π/6)

[tex]\begin{gathered} \sec (x)=\frac{1}{cos(x)} \\ \text{sec}(\frac{7\pi}{6})=\frac{1}{\cos (\frac{7\pi}{6})} \\ \text{sec}(\frac{7\pi}{6})=\frac{1}{-\frac{\sqrt[]{3}}{2}} \\ \text{sec}(\frac{7\pi}{6})=-\frac{2}{\sqrt[]{3}} \end{gathered}[/tex]

Step 5

Rationalize the denominator

[tex]\begin{gathered} \sec (\frac{7\pi}{6})=-\frac{2}{\sqrt[]{3}}\times\frac{\sqrt[]{3}}{\sqrt[]{3}} \\ \sec (\frac{7\pi}{6})=-\frac{2\sqrt[]{3}}{\sqrt[]{9}} \\ \sec (\frac{7\pi}{6})=-\frac{2\sqrt[]{3}}{3} \end{gathered}[/tex]

Hence,

[tex]\sec (\frac{7\pi}{6})=-\frac{2\sqrt[]{3}}{3}[/tex]

I need help with this practice problem It’s asks to Drag the angle measure to each box to match the quadrant location of the terminal ray of the angle op

Answers

Note that the range in quadrants are :

[tex]\begin{gathered} Q1\colon\text{From}\quad 0\pi-0.5\pi \\ Q2\colon\text{From}\quad 0.5\pi-1.0\pi \\ Q3\colon\text{From}\quad 1.0\pi-1.5\pi \\ Q4\colon\text{From}\quad 1.5\pi-2\pi \end{gathered}[/tex]

From the problem,

[tex]\begin{gathered} \frac{3\pi}{4}=0.75\pi\Rightarrow Q2 \\ \frac{57\pi}{8}=7.125\pi \\ \text{Note that 1 whole circle is}\quad 2\pi \\ \text{Subtracting three}\quad 2\pi \\ 7.125\pi-3(2\pi)=1.125\pi \\ \text{and}\quad 1.125\pi\quad \text{ is at Q3} \\ \\ \frac{13\pi}{6}=2.167\pi \\ Subtract\quad 2\pi \\ 2.167\pi-2\pi=0.167\pi\Rightarrow Q1 \end{gathered}[/tex]

The first three answers are :

Q2, Q3 and Q1

For the second set, we have negative angles.

The range of negative angles will be the reversal of the positive angles.

This will be :

[tex]\begin{gathered} Q1\colon\text{From}\quad -1.5\pi\quad to\quad -2\pi \\ Q2\colon\text{From}\quad -1.0\pi\quad to\quad -1.5\pi \\ Q3\colon\text{From}\quad -0.5\pi\quad to\quad -1.0\pi \\ Q4\colon\text{From}\quad -0\pi\quad to\quad -0.5\pi \end{gathered}[/tex]

The following angles are :

[tex]\begin{gathered} -\frac{35\pi}{4}=-8.75\pi \\ \text{Add four}\quad 2\pi \\ -8.75+4(2\pi)=-0.75\pi \\ -0.75\pi\Rightarrow Q3 \\ \\ -\frac{5\pi}{6}=-0.83\pi\Rightarrow Q3 \\ \\ -\frac{5\pi}{11}=-0.45\pi\Rightarrow Q4 \end{gathered}[/tex]

The last three answers are :

Q3, Q3 and Q4

To summarized :

[tex]\begin{gathered} Q1\colon\frac{13\pi}{6} \\ Q2\colon\frac{3\pi}{4} \\ Q3\colon\frac{57\pi}{8},\quad -\frac{35\pi}{4},\quad -\frac{5\pi}{6} \\ Q4\colon-\frac{5\pi}{11} \end{gathered}[/tex]

Sam bought a jacket for $34, which is one-third of the original price. How much did thejacket cost originally?

Answers

Let the original cost of the jacket be x.

Now, saying that one-third of the original cost of the jacket is $34, is mathematically equivalent to:

[tex]\frac{1}{3}\times x=34[/tex]

Now we have to solve the resulting equation in order to obtain the value of x

This is done as follows:

[tex]\frac{1}{3}\times x=34[/tex][tex]\Rightarrow\frac{x}{3}=34[/tex][tex]\Rightarrow x=3\times34[/tex][tex]x=102[/tex]

Therefore, the original cost of the jacket is $102

You have 16 yellow beads, 20 red beads, and 24 orange beads to make identical bracelets. What is the greatest number of bracelets that you can make using all of the beads?

Answers

As per the concept of GCF,  the greatest number of bracelets that you can make using all of the beads is 4.

GCF:

GCF means the largest positive integer not a decimal that divides evenly into all of the numbers in the set. also know as Highest common factor.

Given,

You have 16 yellow beads, 20 red beads, and 24 orange beads to make identical bracelets.

Here we need to find the greatest number of bracelets that you can make using all of the beads.

In order to find it, we have to find the prime factorization of each beads,

So, the prime factorizations of 16, 20 and 24 is,

Factors for 16 is  1, 2, 4, 8, and 16

Factors for 20 is  1, 2, 4, 5, 10, and 20

Factors for 24 is 1, 2, 3, 4, 6, 8, 12, and 24

While we looking into the factors, we have identified that the greatest common factor is 4.

Therefore, there are 4 bracelets that you can make using all of the beads.

To know more about GCF here.

https://brainly.com/question/11444998

#SPJ1

estimate 15.870 + 6.77 by first rounding each number to the nearest tenth

Answers

Given:

15.870 + 6.77

We are required to round each number to the nearest tenth before performing the addition.

The tenth digit is the number first number after the decimal point.

First step:

Round to the nearest tenth

15.870 ==> 15.9

6.77 ==> 6.8

Second step:

Add both numbers after rounding to the nearest tenth

15.9 + 6.8 = 22.7

ANSWER:

22.7

2) A taxi service charges a fee of $2.50 and then an additional $2.70 per mile. Determine the relationship.

Answers

Data:

• Fixed fee: $2.50

,

• Additional: $2.70 per mile

Procedure:

The relationship the problem is describing is a linear one with the form:

[tex]y=mx+b[/tex]

In this case, we have a fixed fee that is represented by b in the linear equation, and an additional fee represented by m, which depends on the miles (x) travels.

Thus, the equation would be:

[tex]y=2.7x+2.5[/tex]

Based on this equation, you can replace any value of miles (x) given to calculate the total price (y).

I am going to send the pictures Please solve question b(b) Canada like many countries use the metric system if the Canada news says it’s-2 degrees Celsius what is that in Fahrenheit ( Celsius is typically rounded to the tenth place

Answers

Given equation:

[tex]\text{ Wind Chill = }35.74\text{ }+\text{ }0.6215T\text{ }-\text{ 35}.75(V^{0.16})\text{ }+\text{ }0.4275T(V^{0.16})[/tex]

Where T = temperature in Fahrenheit and

V = wind speed in miles per hour

Conversion formulas:

[tex]\begin{gathered} A_s\text{ = }M_s\text{ }\times\text{ }0.62 \\ F\text{ = 1.8C + 32} \end{gathered}[/tex]

Question (b)

We are required to convert -2 degree Celsius to Fahrenheit

Using the conversion formula:

[tex]\begin{gathered} F\text{ = }1.8\text{ }\times-2\text{ + 32} \\ =\text{ 28.4} \end{gathered}[/tex]

Answer: 28.4 F

Find the arc-length of the sector of a circle with the given radius r and central angle 0. Give the answer in the given unit of measure, rounded to the nearest hundredth.r = 25 m; θ = 12π/7

Answers

Given a radius and a central angle in radian, the formula in solving arc length is:

[tex]AB=\theta\times r[/tex]

Since the angle and radius are given already in the problem, let's plug it in to the formula above.

[tex]AB=\frac{12\pi}{7}(25m)[/tex]

Then, solve.

[tex]AB=\frac{300\pi m}{7}=\frac{942.47779m}{7}\approx134.64m[/tex]

Therefore, the length of an arc having a central angle of 12π/7 is approximately 134.64 meters.

Identify the sampling technique used (or to be used) in the following scenarios. Possible answers could be simple random, systematic random or stratified random sampling.

Answers

a)

This sampling technique is simple random sampling, since the students are selected at random by drawing their names from a piece of paper.

b)

Since there is a condition to select a resident (being the sixth resident of the list), this technique is classified as systematic random sampling.

c)

The 20 students were selected by using a table of random numbers without any criteria. Therefores, this is a simple random sampling technique.

d)

Since, among the 100 selected random people, there was a pre determined subgroup of 5 barangays, this is a stratified random sampling technique.

e)

Since everyone in the sample was selected at random by dwaring lots, this is a simple random sampling technique.

(Lesson 9.8: Other Steady-State Methods.) Consider the following observations: 54 70 75 62 If we choose a batch size of 3, calculate all of the overlapping batch means for me. a. 65.25 b. 62.0, 68.5 c. 66.3, 69.0 d. 65.25 +3 e. None of the above

Answers

All of the overlapping batch means for me is (66.3, 69.0).

Given observations:

54 70 75 62

From the given data, if we choose a batch size of 3, then calculating all of the overlapping batch means for me in total

that is :

= 1/3 * (54 + 70 + 75)

= 66.3

and

= 1/3 * (70 + 75 + 62)

= 69.0

So often, the option c is correct from the given

that is 66.3, 69.0

Hence the answer is all of the overlapping batch means for me is (66.3, 69.0).

To learn more about observations click here https://brainly.com/question/24343193

#SPJ4

if i did
what would i get?

Answers

?

be more specific :)

1. Mr. and Mrs. Ryan Miller bought a
refrigerator for $1,416. They agreed to
make 12 equal monthly payments. How
much more than $50 will each payment
be?
2. The $1,416 paid by the Millers (problem 1
to buy the refrigerator included an interest
charge of $188. What was the cash cost of
the refrigerator?

Answers

The monthly payment more than $50 is $68 and the original price of the refrigerator is $1228

Mr. and Mrs. Ryan Miller bought a refrigerator for $1,416.

They agreed to make 12 equal monthly payments

Monthly payment = 1416 / 12

The monthly payment is $118

more than $50 will each payment be = 118 - 50 = 68

The $1,416 paid by the Miller to buy the refrigerator included an interest charge of $188

The original price is 1416 - 188 = 1228

Therefore, the monthly payment more than $50 is $68 and the original price of the refrigerator is $1228

To learn more about word problem refer here

https://brainly.com/question/13818690

#SPJ9

frequency The table and corresponding polygon show information about the waiting times of some patients at a dentist. Frequency What fraction of patients waited for more than 7 minutes? 10- 0- 5 6 7 8 Waiting time (x minutes) 9 10 x Waiting time (x minutes) 5< x≤6 6< x≤7 7< x≤8 8< x≤9 9< x≤ 10​

Answers

The 31 minutes for the lowest quartile of the waiting times of patients at this surgery.

Given that,

The histogram displays statistics about the average patient wait time in dental offices, expressed in minutes. 3.24 frequency The table and related polygon display data regarding some patients' wait periods at a dentist.

We have to find frequency How many patients waited for longer than seven minute.

The frequency of a given data value is the number of times it happens. We use f to represent a data value's frequency. For instance, if five students received As in science, the grade A was said to have a frequency of five.

In the picture we can see the answer.

Therefore, The 31 minutes for the lowest quartile of the waiting times of patients at this surgery.

To learn more about minutes visit: https://brainly.com/question/15600126

#SPJ9

Which inequality in factored form represents the region less than the quadratic function with zeros-40 and -50 and
includes the point (-55, -75) on the boundary line?
O y<-(x-40)(x-50)
O ys-(x+40)(x+50)
Oys-(x-40)(x - 50)
O y<-(x +40)(x+50)



Please help

Answers

The inequality that reflects the given region, according to the Factor Theorem, is:

y< -(x+40)(x+50)

What is the Factor Theorem?

When completely factoring polynomials, the factor theorem is employed in mathematics. It is a theorem that relates the factors and zeros of a polynomial. If f(x) is a polynomial of degree n 1 and 'a' is any real number, then (x-a) is a factor of f(x) if f(a)=0.

According to the Factor Theorem, a polynomial function with roots x₁, x₂, ....xₙ is given by

f(x)=a(x-x₁)(x-x₂)...(x-xₙ)

In which a is the leading coefficient.

The roots are given as follows:

x₁=-40, x₂=-50

Hence:

y = a(x + 40)(x +50)

It includes the point (-55,-75), hence:

-75 = a(-55 + 40)(-55 +50)

a = 75/(15 x 5)

a = 1

The equation that is less than the region is:

y< -(x+40)(x+50)

To know more about factor theorem, visit:

https://brainly.com/question/13547729

#SPJ1

Answer:

d

Step-by-step explanation:

Determine the solution to the inequality. |4x − 4| ≥ 8 x ≤ −1 or x ≥ 3 x ≤ −2 or x ≥ 3 x ≤ −3 or x ≥ 4 x ≤ −4 or x ≥ 4

Answers

The solution to the inequality will be -

x ≥ 3    or   x ≤ -1

What is an Inequality? What is a expression? What is a mathematical equation?An inequality in mathematics compares two values or expressions, showing if one is less than, greater than, or simply not equal to another value.A mathematical expression is made up of terms (constants and variables) separated by mathematical operators.A mathematical equation is used to equate two expressions. Equation modelling is the process of writing a mathematical verbal expression in the form of a mathematical expression for correct analysis, observations and results of the given problem.

We have the given inequality as -

|4x − 4| ≥ 8

We have the inequality as -

|4x − 4| ≥ 8

4x - 4 ≥ 8     or     4x - 4 ≤ - 8  

4x ≥ 12      or      4x ≤ - 4

x ≥ 3    or   x ≤ -1

Therefore, the solution to the inequality will be -

x ≥ 3    or   x ≤ -1

To solve more questions on inequalities, visit the link below -brainly.com/question/1782515

#SPJ1

pls help guyssssssssss

Answers

There is nothing here

Biologists notice an alarming trend in the walleye population in Lake Erie. Despite efforts, the population is decreasing by 15% each year. They estimate the walleye population was 26 million at the beginning of the current year. Write a sequence that lists the population for the first 4 years using 26 million as the first term.

Answers

Given:

The initial population is p = 26 million.

The decreasing percentage is r = 15% each year = 0.15.

The objective is to write the sequence of population after 4 years.

Explanation:

The general formula of population decrease is,

[tex]A=P(1-r)^t\text{ . . . . .(1)}[/tex]

The population in the 1st-year can be calculated by substituting t=1 in equation (1).

[tex]\begin{gathered} A_1=26(1-1.15)^1 \\ =26(0.85) \\ =22.1\text{million} \end{gathered}[/tex]

The population in the 2nd year can be calculated by substituting t=2 in equation (1).

[tex]\begin{gathered} A_2=26(1-0.15)^2 \\ =26(0.85)^2 \\ =18.785million \end{gathered}[/tex]

The population in the 3rd year can be calculated by substituting t=3 in equation (1).

[tex]\begin{gathered} A_3=26(1-0.15)^3 \\ =26(0.85)^3 \\ =15.967255million \end{gathered}[/tex]

Hence, the population for the first 4 years will be,

26million,

22.1million,

18.785million,

15.967255million.

PLEASE HELP IT'S DUE NOW.. :(

Answers

Slope intercept form of equation of line f and line g is

Equation of line f

y = 1.75x + 3.5

Equation of line g

y = -4x - 8

First option is correct

What is equation of line in slope intercept form?

The most general form of equation of line in slope intercept form is given by y = mx + c

Where m is the slope of the line and c is the y intercept of the line.

Slope of a line is the tangent of the angle that the line makes with the positive direction of x axis.

If  [tex]\theta[/tex] is the angle that the line makes with the positive direction of x axis, then slope (m) is given by

m =  [tex]tan\theta[/tex]

The distance from the origin to the point where the line cuts the x axis is the x intercept of the line.

The distance from the origin to the point where the line cuts the y axis is the y intercept of the line.

Here,

For line f

The line passes through (2, 7) and (4, 10.5)

Slope =

[tex]\frac{10.5 - 7}{4 - 2}\\\frac{3.5}{2}\\\frac{7}{4}[/tex]

Equation of line =

[tex]y - 7 = \frac{7}{4}(x - 2)[/tex]

y = 1.75x - 3.5+7

y = 1.75x  + 3.5

For line g

The line passes through (-3, 4) and (-2, 0)

Slope =

[tex]\frac{0-4}{-2-(-3))}\\-4[/tex]

Equation of line =

y - 4 = -4(x - (-3))

y - 4 = -4x-12\\

y =-4x-12+4\\

y = -4x-8

The first option is correct

To learn more about equation of line in slope intercept form, refer to the link-

brainly.com/question/25514153

#SPJ1

A circular dartboard has diameter 40cm. Its bull’s eye has diameter of 8 cm. if an amateur throws a dart and it hits the board, what is the probability that the dart hits the bull’s eye.

Answers

Answer:

a value of is required in the following exercises, use

A circular dartboard has diameter 40 Its bull's eye has diameter 8

a. If an amateur throws a dart and it hits the board. What is the probability that the dart hits the bull's eye?

b. After many throws, 75 darts have hit the target. Estimate the number hitting the bull's eye.

Step-by-step explanation:

hope it helps! please mark brainlets

Express 72 1/2% asa fraction in its lowest term

Answers

The percentage of the number is 29/40 when The number is 72 1/2.

Given that,

The number is 72 1/2

We have to find the percentage of the number.

The Latin word "per centum," which means "by the hundred," is where the word "percentage" originally came from. Percentages are fractions when the denominator is 100. To put it another way, it's the relationship between a part and a whole in which the value of the whole is always assumed to be 100.

We have number,

72 1/2

145/2

145/2× 1/100

145/200

29/40

Therefore, The percentage of the number is 29/40 when The number is 72 1/2.

To learn more about percentage visit: https://brainly.com/question/14508634

#SPJ9

Cole scored 18 points in his team's last game. That was 36% of the team's total points for the
game. How many total points did the team score?

Answers

The team's total points for the game was 50.

According to the question,

We have the following information:

Cole scored 18 points in his team's last game. That was 36% of the team's total points for the game.

Now, let's take the score points of the team in the game to be x points.

So, we have the following expression:

36% of x = 18

36x/100 = 18

Multiplying by 100 on both sides of the equation:

36x = 1800

Dividing by 36 on both sides of the equation:

x = 1800/36

x = 50

Hence, the team's total points for the game was 50.

To know more about team's total point here

https://brainly.com/question/22666623

#SPJ1

Which relation is a function?
I’m confused on this. can anyone help me out?

Answers

Answer:

The one the u have marked is a function.

Step-by-step explanation:

If you draw a vertical line through each one, each line only goes through one point

You want to order posers to advertise your band. A company charges $109.95 for the first 100 posters and $65 for each additional 100 posters.
Write an equation that represents the cost (in dollars) of the posters of the number (in hundreds) of posters ordered (in slope- intercept form).

Answers

The equation that represents the total cost as a function of the number (in hundreds) of posters ordered, x is f(y) = 109.95 + 65x

let

y = total cost

x = numbers in hundreds

cost of first hundred = $109.95

Cost of additional hundred = $65

f(y) = 109.95 + 65x

The total cost of 1000 posters

1000 - first hundred = 900

additional hundreds = 900/100= 9

So,

f(y) = 109.95 + 65x

= 109.95 + 65(9)

= 109.95 + 585

= 694.95

f(y) = $694.95

Therefore, the total cost of 1000 posters is f(y) = $694.95

Learn more about linear function refer here

brainly.com/question/13136492

#SPJ1

Isosceles triangle JKL has a perimeter of 36 units and the given vertices.

J (-3, -9)

K (-3, 7)

L (X, -1)

What is the possible x-coordinate for point L?

Answers

The possible x-coordinate for point L are x = (-16.85)(10.85)

What is perimeter?

For geometry, the perimeter of the shape is defined as the total length of the boundary. The perimeter is determined by adding all the sides and side lengths that enclose the shape. It is measured in units such as centimeters, meters, feet and inches,

For the given question:

JK = [tex]\sqrt{(-3-(-3))^{2}+(-9-7)^{2} }[/tex]

JK = [tex]\sqrt{0^{2} +16^{2} }[/tex]

JK = 16

JL = [tex]\sqrt{(-3-x)^{2} +(-9-(-1))^{2} }[/tex]

JL = [tex]\sqrt{(-3-x)^{2} + 8^{2} }[/tex]

JL = [tex]\sqrt{(-3-x)^{2} + 64}[/tex]

KL = [tex]\sqrt{(-3-x)^{2} + (7-(-1))^{2} }[/tex]

KL = [tex]\sqrt{(-3-x)^{2} + 8^{2} }[/tex]

KL = [tex]\sqrt{(-3-x)^{2} + 64}[/tex]

Since, JK + JL+ KL = 36 [Perimeter of triangle]

As we have already calculated, JK = 16

So, 16 + JL+ KL = 36

JL+ KL = 36 - 16

JL+ KL = 20

Since  JKL is an isosceles triangle, JL= KL

So, let's replace KL by JK in the above equation:

JL+ JL = 20

2JL = 20

JL = 10

Now, substitute the value of JL in following equation:

JL = [tex]\sqrt{(-3-x)^{2} + 64}[/tex]

16 =  [tex]\sqrt{(-3-x)^{2} + 64}[/tex]

Now square both sides,

256 = (-3-x)² + 64

256-64 = (-3-x)²

192 = (-3-x)²

13.85 =  -3-x

x = -3 ± 13.85

x = (-3 -  13.85) (-3 + 13.85)

x = (-16.85)(10.85)

To know more about perimeter visit:

https://brainly.com/question/27591380

#SPJ1

Mr. Jones' age is 3 years more than 4 times Mahelet's age. If the sum of their ages is 73, how old is Mahelet?

Answers

Mahelet's age is found to be 14 years by solving the simultaneous equations using the given data.

What exactly is a simultaneous equation?

A collection of two or more equations, each having two or more variables, whose values can concurrently fulfil one, more, or all of the equations in the collection, with the number of variables being equal to or fewer than the collection's equations.

Given: Mahelet is four times older than Mr. Jones, who is three years older than Mahelet. The total of their ages is 73.

Let, x =  Mahelet's age

      y =  Mr. Jones's age

We know that,

y = 4x +3

x + y = 73

Solving these equations simultaneously we get,

y = 4(73-y) +3

y = 292 - 4y + 3

5y = 295

y = 59

x = 73 - y = 73 - 59 = 14

Therefore, Mahelet's age is found to be 14 years by solving the simultaneous equations.

Learn more about simultaneous equations here:

https://brainly.com/question/15165519

#SPJ1

If u = 1 + 3i and v = -2 − i, what is u + v?

Answers

Answer:

2i - 1

Step-by-step explanation:

The expression is,

→ u + v

Simplifying the expression,

→ u + v

→ (1 + 3i) + (-2 - i)

→ (3i - i) + (1 - 2)

→ 2i - 1

Hence, the answer is 2i - 1.

Step-by-step explanation:

you need to replace definition of both u and v into the equation

u + v = (1+3i) + (-2-i)

= 1 + 3i -2 - i

= 3i - i + 1 - 2

= 2i - 1

During a snowstorm, Amelia tracked the amount of snow on the ground.When the storm began, there was 1 inch of snow on the ground. For the first 2hours of the storm, snow fell at a constant rate of 3 inches per hour. Thestorm then stopped for 3 hours and then started again at a constant rate of 1inch every 2 hours for the next 6 hours. Make a graph showing the inches ofsnow on the ground over time using the data that Amelia collected.

Answers

First, let's write on a table the information about the snow on the ground for the 11 hours it snowed:

period of time in hours amount of snow in inches

0 1

from 0 to 2 1 + t * 3 (initial amount plus 3 in/h * number of hours)

from 2 to 5 1 + 2*3 = 7 (the same amount as when it stopped raining)

from 5 to 11 7 + (t-5)/2 (amount at t = 5 plus 1 in/2h * number of

hours since hour 5)

Now, using this information on a graph, we obtain:

please help NEED FAST

Answers

a) The quadratic equation behind the parabola is y = (4 / 5) · x² - (8 / 5) · x - 1.

b) There are two x-intercepts: x₁ = - 0.5, x₂ = 2.5.

How to derive a quadratic equation and find its x-intercepts

Mathematically speaking, parabolas are represented by quadratic equations, whose standard form is introduced below:

y = a · x² + b · x + c

Where a, b, c are real coefficients.

The values of the three coefficients are found from the knowledge of three distinct points on Cartesian plane. First, choose the three points:

(x₁, y₁) = (- 0.5, 0), (x₂, y₂) = (2.5, 0), (x₃, y₃) = (0, - 1)

Second, construct the system of linear equations with all the given points and the standard form of the quadratic equation:

0.25 · a - 0.5 · b + c = 0

6.25 · a + 2.5 · b + c = 0

c = - 1

Third, solve the system by numerical methods:

(a, b, c) = (4 / 5, - 8 / 5, - 1)

Fourth, write the quadratic equation:

y = (4 / 5) · x² - (8 / 5) · x - 1

The x-intercepts of the quadratic equation are the points of the curve that pass through the x-axis. Then, the x-intercepts are x₁ = - 0.5 and x₂ = 2.5.

To learn more on quadratic equations: https://brainly.com/question/1863222

#SPJ1

Other Questions
Use the Distributive Property solve the equation.- 6(x + 3) = 30 how do you do this why does it make me put so many words Team A, B, and C are competing in a basketball tournament. The probability of team A winning is 0.2, the probability of team B winning is 0.45, and the probability of team C winning is 0.35. Anna can join either team A or team B. Elina can join either team B or team C. Nancy can join either team A or team C. Who is most likely to win?AnnaNancyElenaAll have an equal probability to win. Which relation represents a function sample A or sample B If you receive 360 promotional emails per month and only 2.5% percent of those emails are of interest to you, what is the expected number of promotional emails that will be of interest to you each month? graph the quadrilateral with the given vertices in a coordinate plane. then show the quadrilateral in a a parallelogram A resident is to receive 2 ounces of liquid . You know 30 ccs equals one ounce . How many ccs of the liquid will you give to ensure the resident receives 2 ounces Which of the following is considered a fall prevention system because it stops a person from falling in the first place?Select the best option.AnchorGuardrailsPersonal fall arrest systemsSafety nets 1.4 / 31.5 The Quotient is ______ 210000The first five multiples for the numbers 4 and 6 are shown below.Multiples of 4: 4, 8, 12, 16, 20, ...Multiples of 6: 6, 12, 18, 24, 30,...What is the least common multiple of 4 and 6?222122423 24 25Mark this and returnSave and ExitNext equivalent of it is not true that the test is today or the party is tonight A baseball player throws a ball horizontally off a cliff that is 19.8 m tall. The ball lands 29.3 m away from the base of the cliff. What is the horizontal velocity of the ball? Which of the following equations is equivalent to the given equation? nQuestion 6Mutiple Choice Worth 1 points)(06.04 MC)The length of a rectangle is represented by the function L(x)= 2x. The width of that same rectangle is represented by the function W(x)=8x-4x+1. Which of the following shows the area of the rectangle interms of x?(L+ W)(x)=8x-2x+1(L + W)(x)=8x - 6x +1(L W)(x)=16x-4x+1(L W)(x)=16x-8x+2x if I need 1/2 cup of oil but I only have 1/3 cup of oil how much oil do I need need help asappppppp _________ are groups of organisms with the same genetic code that breed and produce offspring that can breed. AdaptationSpeciesSelectionCommunity Dont know how to solve with the -1 before the x lebron walked 4 1/2 miles to library in 2 1/4 hours. he walked the return trip at the same average rate , but a different route, taking my 2 1/2 hours. How many miles did lebron walk on the return trip? Use the quadratic formula to solve for X. 3x^2 = -3x +7